算术级数中连续素数之间的差异-MathOverflow mathoverflow.net上最近的30个 2024-06-29T02:01:17Z年 https://mathoverflow.net/feeds/question/423247 https://creativecommons.org/licenses/by-sa/4.0/rdf https://mathoverflow.net/q/423247 9 算术级数中连续素数之间的差异 埃里克·纳斯隆德 https://mathoverflow.net/users/12176 2022-05-24T16:32:47Z 2022-05-25T12:00:49Z <p>让$\pi(x)=\sum_{p\leqx}$</span>表示素数计数函数<a href=“https://londmathsoc.onlinelibrary.wiley.com/doi/abs/10.112/plms/83.3532“rel=”nofollow noreferrer“>Baker、Harman和Pintz关于素数间隙的一个众所周知的结果表明,对于$x\geqy\geqx^{0.525}$</span>,我们得到了</p><p><span class=“math-container”>$$\pi(x+y)-\ pi(x)\gg\frac{y}{\logx}$$</span></p><p>设$\pi(x;q,a)$</span>表示素数$p\leqx$</sspan>,使得$p\equiva\pmod{q}$</spa>。这个定理对$\pi(x;q,a)$</span>有类似的版本吗</p><区块报价><p>问题:最小的增长函数是什么?对于任何<span class=“math-container”>$x\geqy\geqf(x,q)$</span>,我们有$$\pi(x+y;q,a)-\pi$$</span></p></blockquote><p><a href=“的推论13.8https://www.cambridge.org/core/books/amplicative-number-theory-i/4E45519B26115AEEA4839C6C38206ACD“rel=”nofollow noreferrer“>Montgomery和Vaughn的乘数理论指出,在广义黎曼假设下,$$\psi(x;q,a)=\sum_{begin{array}{c}n \leq x号\\n等于a(q)\结束{数组}}\Lambda(n)=\frac{x}{\phi(q)}+O\left(x^{1/2}\log^{2} x个\右侧)$$</span></p><p>根据这个推论,可以得出在GRH下,$$f(x,q)=\phi(q)x^\frac{1}{2}\log^{2}(x)的期望下限成立$$</span></p><p>我知道<a href=“https://link.springer.com/article/10.1007/BF01188631“rel=”nofollow noreferrer“>1933年Heilbronn(MR1545353)的结果,我认为(我的德语不太好)证明了对于固定的$q,$</span>,存在$\delta&gt;0$,因此对于$x\geqy\geqx^{1-\delta}$</sspan>数学容器“>$$\pi(x+y;q,a)-\pi(x;q,a)\sim\frac{y}{\phi(q)\log x}。$$</span></p><p>关于这个话题有什么最新的结果吗</p> https://mathoverflow.net/questions/423247/-/423292#423292 5 由2734364041回答算术级数中连续素数之间的差异 2734364041 https://mathoverflow.net/users/111215 2022-05-25T12:00:49Z 2022-05-25T12:00:49Z <p>无条件地,可以取$f(x,q)=x^{0.525}$</span>,前提是$q$在Siegel-Walfisz范围内。存在一个(适当小的)常数$\delta&gt;0$</span>,这样就可以对除$q\leqx^{delta}$</sspan>的密度零子集以外的所有对象取$f(x,q)=x^{0.525}$</span</p><p>这源于Bombieri-Vinogradov型结果,该结果为素数指示函数精心构建的下限(由于<a href=“https://academicial.oup.com/qjmath/article/53/4/479/1537722?searchresult=1“rel=”noreferrer“>Kumchev</a>,但将$0.525$替换为$0.53$(由于<a href=“math-container”>$L(s,\chi)^2$</span>乘以Dirichlet多项式的二阶矩https://academic.oup.com/qjmath/article/55/3/307/1577135“rel=”noreferrer“>Harman、Watt和Wong)。有关如何将这两个结果结合起来的详细信息,请参阅<a href=“https://link.springer.com/article/10.1007/s40993-018-0109-y“rel=”noreferrer“>Alweiss和Luo(见其中的定理2.3)。然后取$Q=(\log x)^a$</span></p>